Sum of fields is never a field

  • Thread starter Thread starter rainiscoming
  • Start date Start date
  • Tags Tags
    Field Fields Sum
Click For Summary
The discussion focuses on proving that the direct sum of two or more fields, denoted as F × G, is not a field. Participants emphasize that F × G must satisfy specific properties to be classified as a field, including the existence of multiplicative inverses. A key point raised is the challenge of proving that the product of elements in F × G has an inverse within the set, particularly when the product results in (a1a2, b1b2), which may not be invertible. The conversation highlights the need to disprove at least one field property to establish that F × G cannot be a field. Ultimately, the participants are working through the implications of these properties to reach a conclusion about the nature of direct sums of fields.
rainiscoming
Messages
1
Reaction score
0

Homework Statement


Prove that a direct sum of two or more field is never a field


Homework Equations



F X G = {(f,g): f in F, g in G}

The Attempt at a Solution



I know that I need to prove FXG is Abelian group under addition, and FXG - {0,0} is an Abelian group under mult.
And for mult,
I know I need to check 1) mult identity, 2) closure, 3) commutative, 4) mult inverse, 5) associativity

I have problem proving mult inverse,

let a1, a2, b1, b2 be elements in F XG - {0,0}, such that (a1,b1)*(a2,b2) = (a1a2, b1b2)
so (a1a2, b1b2) ^-1 = (1/(a1a2), 1/(b1b2)) which is not in FXG - (0,0)

Is this right?
 
Last edited:
Physics news on Phys.org
How do you know a1a2 is invertible?
 
Remember you're trying to prove that F\times G is not a field.

So you're not actually trying to prove any of the things you say you're trying to prove. You're trying to disprove at least one of them. So if you've had a problem proving one of them, that might give you a clue.
 
Question: A clock's minute hand has length 4 and its hour hand has length 3. What is the distance between the tips at the moment when it is increasing most rapidly?(Putnam Exam Question) Answer: Making assumption that both the hands moves at constant angular velocities, the answer is ## \sqrt{7} .## But don't you think this assumption is somewhat doubtful and wrong?

Similar threads

Replies
10
Views
3K
  • · Replies 12 ·
Replies
12
Views
2K
  • · Replies 7 ·
Replies
7
Views
14K
Replies
1
Views
2K
  • · Replies 17 ·
Replies
17
Views
3K
  • · Replies 1 ·
Replies
1
Views
1K
  • · Replies 1 ·
Replies
1
Views
2K
  • · Replies 8 ·
Replies
8
Views
2K
  • · Replies 8 ·
Replies
8
Views
2K
  • · Replies 2 ·
Replies
2
Views
2K